A maximal cardinality subset of n lattice points so that all points in the subset have distance at least 4












1














If we have some random set of $n$ lattice points what is the maximum cardinality of a subset in which all points have distance at least $4$ (or some other number).
I really hope the best bound is not the trivial $n/16$. I'm not asking for a proof that this bound is the best possible, just provide the best bound you can.










share|cite|improve this question





























    1














    If we have some random set of $n$ lattice points what is the maximum cardinality of a subset in which all points have distance at least $4$ (or some other number).
    I really hope the best bound is not the trivial $n/16$. I'm not asking for a proof that this bound is the best possible, just provide the best bound you can.










    share|cite|improve this question



























      1












      1








      1


      1





      If we have some random set of $n$ lattice points what is the maximum cardinality of a subset in which all points have distance at least $4$ (or some other number).
      I really hope the best bound is not the trivial $n/16$. I'm not asking for a proof that this bound is the best possible, just provide the best bound you can.










      share|cite|improve this question















      If we have some random set of $n$ lattice points what is the maximum cardinality of a subset in which all points have distance at least $4$ (or some other number).
      I really hope the best bound is not the trivial $n/16$. I'm not asking for a proof that this bound is the best possible, just provide the best bound you can.







      combinatorics combinatorial-geometry extremal-combinatorics






      share|cite|improve this question















      share|cite|improve this question













      share|cite|improve this question




      share|cite|improve this question








      edited Dec 4 '18 at 6:16









      Alex Ravsky

      39.2k32080




      39.2k32080










      asked Dec 3 '18 at 8:26









      mr. clock

      203




      203






















          2 Answers
          2






          active

          oldest

          votes


















          0














          I’ll try to provide a framework for the question. Let $d$ be a fixed distance $d$ (equal to $4$ in our case).
          A set $A’$ of lattice points we shall call $d$-separated, if any two distinct points of $B$ are placed at distance at least $d$. We are interested in a smallest $r=r(d)$ such that any finite set $A$ of lattice points has a $d$-separated subset $B$ of size at least $|A|/r$. I guess Empy2 obtained a bound $rle 15$ by constructing a coloring of $Bbb Z^2$ into $15$ colors such that each monochromatic subset is $d$-separated. Indeed, in this case as $B$ we can chose a largest monochromatic subset of $A$, which imply $|B|ge |A|/15$ . Unfortunately, it can be easily shown that there is no such coloring in $14$ colors. (I’m going to write a proof later.) But this not imply that $r>14$, because a $d$-separated subset $B$ of $A$ may be chosen by different method. So there is sense to look for subsets $A$ of lattice points with the biggest ratio $|A|/|B|$, where $B$ is a maximal $d$-separated subset of $A$.



          The following set has ratio $14$.



          ...
          ....
          ....
          ...


          Are there subsets of lattice points with the bigger ratio?






          share|cite|improve this answer





























            1














            From the $ntimes n$ array, the following gives about $ n^2/15$, or one point in every fifteen.



            $$x(3,3)+y(-1,4)$$






            share|cite|improve this answer



















            • 1




              Do you think it's possible to achieve $n^2/14$? That was actually a bound I was hoping for to solve a problem which led me to this question. The problem says that it's always possible to choose a subset of a set of unit circles having area $S$ so that the area of a chosen subset is at least $2S/9$. If you use a well-known result saying that for a figure of area $S$ it's always possible to set a coordinate system so that the figure contains at least $S$ lattice points it quickly becomes clear that a bound of $n^2/14$ would solve the problem.
              – mr. clock
              Dec 4 '18 at 8:48











            Your Answer





            StackExchange.ifUsing("editor", function () {
            return StackExchange.using("mathjaxEditing", function () {
            StackExchange.MarkdownEditor.creationCallbacks.add(function (editor, postfix) {
            StackExchange.mathjaxEditing.prepareWmdForMathJax(editor, postfix, [["$", "$"], ["\\(","\\)"]]);
            });
            });
            }, "mathjax-editing");

            StackExchange.ready(function() {
            var channelOptions = {
            tags: "".split(" "),
            id: "69"
            };
            initTagRenderer("".split(" "), "".split(" "), channelOptions);

            StackExchange.using("externalEditor", function() {
            // Have to fire editor after snippets, if snippets enabled
            if (StackExchange.settings.snippets.snippetsEnabled) {
            StackExchange.using("snippets", function() {
            createEditor();
            });
            }
            else {
            createEditor();
            }
            });

            function createEditor() {
            StackExchange.prepareEditor({
            heartbeatType: 'answer',
            autoActivateHeartbeat: false,
            convertImagesToLinks: true,
            noModals: true,
            showLowRepImageUploadWarning: true,
            reputationToPostImages: 10,
            bindNavPrevention: true,
            postfix: "",
            imageUploader: {
            brandingHtml: "Powered by u003ca class="icon-imgur-white" href="https://imgur.com/"u003eu003c/au003e",
            contentPolicyHtml: "User contributions licensed under u003ca href="https://creativecommons.org/licenses/by-sa/3.0/"u003ecc by-sa 3.0 with attribution requiredu003c/au003e u003ca href="https://stackoverflow.com/legal/content-policy"u003e(content policy)u003c/au003e",
            allowUrls: true
            },
            noCode: true, onDemand: true,
            discardSelector: ".discard-answer"
            ,immediatelyShowMarkdownHelp:true
            });


            }
            });














            draft saved

            draft discarded


















            StackExchange.ready(
            function () {
            StackExchange.openid.initPostLogin('.new-post-login', 'https%3a%2f%2fmath.stackexchange.com%2fquestions%2f3023777%2fa-maximal-cardinality-subset-of-n-lattice-points-so-that-all-points-in-the-subse%23new-answer', 'question_page');
            }
            );

            Post as a guest















            Required, but never shown

























            2 Answers
            2






            active

            oldest

            votes








            2 Answers
            2






            active

            oldest

            votes









            active

            oldest

            votes






            active

            oldest

            votes









            0














            I’ll try to provide a framework for the question. Let $d$ be a fixed distance $d$ (equal to $4$ in our case).
            A set $A’$ of lattice points we shall call $d$-separated, if any two distinct points of $B$ are placed at distance at least $d$. We are interested in a smallest $r=r(d)$ such that any finite set $A$ of lattice points has a $d$-separated subset $B$ of size at least $|A|/r$. I guess Empy2 obtained a bound $rle 15$ by constructing a coloring of $Bbb Z^2$ into $15$ colors such that each monochromatic subset is $d$-separated. Indeed, in this case as $B$ we can chose a largest monochromatic subset of $A$, which imply $|B|ge |A|/15$ . Unfortunately, it can be easily shown that there is no such coloring in $14$ colors. (I’m going to write a proof later.) But this not imply that $r>14$, because a $d$-separated subset $B$ of $A$ may be chosen by different method. So there is sense to look for subsets $A$ of lattice points with the biggest ratio $|A|/|B|$, where $B$ is a maximal $d$-separated subset of $A$.



            The following set has ratio $14$.



            ...
            ....
            ....
            ...


            Are there subsets of lattice points with the bigger ratio?






            share|cite|improve this answer


























              0














              I’ll try to provide a framework for the question. Let $d$ be a fixed distance $d$ (equal to $4$ in our case).
              A set $A’$ of lattice points we shall call $d$-separated, if any two distinct points of $B$ are placed at distance at least $d$. We are interested in a smallest $r=r(d)$ such that any finite set $A$ of lattice points has a $d$-separated subset $B$ of size at least $|A|/r$. I guess Empy2 obtained a bound $rle 15$ by constructing a coloring of $Bbb Z^2$ into $15$ colors such that each monochromatic subset is $d$-separated. Indeed, in this case as $B$ we can chose a largest monochromatic subset of $A$, which imply $|B|ge |A|/15$ . Unfortunately, it can be easily shown that there is no such coloring in $14$ colors. (I’m going to write a proof later.) But this not imply that $r>14$, because a $d$-separated subset $B$ of $A$ may be chosen by different method. So there is sense to look for subsets $A$ of lattice points with the biggest ratio $|A|/|B|$, where $B$ is a maximal $d$-separated subset of $A$.



              The following set has ratio $14$.



              ...
              ....
              ....
              ...


              Are there subsets of lattice points with the bigger ratio?






              share|cite|improve this answer
























                0












                0








                0






                I’ll try to provide a framework for the question. Let $d$ be a fixed distance $d$ (equal to $4$ in our case).
                A set $A’$ of lattice points we shall call $d$-separated, if any two distinct points of $B$ are placed at distance at least $d$. We are interested in a smallest $r=r(d)$ such that any finite set $A$ of lattice points has a $d$-separated subset $B$ of size at least $|A|/r$. I guess Empy2 obtained a bound $rle 15$ by constructing a coloring of $Bbb Z^2$ into $15$ colors such that each monochromatic subset is $d$-separated. Indeed, in this case as $B$ we can chose a largest monochromatic subset of $A$, which imply $|B|ge |A|/15$ . Unfortunately, it can be easily shown that there is no such coloring in $14$ colors. (I’m going to write a proof later.) But this not imply that $r>14$, because a $d$-separated subset $B$ of $A$ may be chosen by different method. So there is sense to look for subsets $A$ of lattice points with the biggest ratio $|A|/|B|$, where $B$ is a maximal $d$-separated subset of $A$.



                The following set has ratio $14$.



                ...
                ....
                ....
                ...


                Are there subsets of lattice points with the bigger ratio?






                share|cite|improve this answer












                I’ll try to provide a framework for the question. Let $d$ be a fixed distance $d$ (equal to $4$ in our case).
                A set $A’$ of lattice points we shall call $d$-separated, if any two distinct points of $B$ are placed at distance at least $d$. We are interested in a smallest $r=r(d)$ such that any finite set $A$ of lattice points has a $d$-separated subset $B$ of size at least $|A|/r$. I guess Empy2 obtained a bound $rle 15$ by constructing a coloring of $Bbb Z^2$ into $15$ colors such that each monochromatic subset is $d$-separated. Indeed, in this case as $B$ we can chose a largest monochromatic subset of $A$, which imply $|B|ge |A|/15$ . Unfortunately, it can be easily shown that there is no such coloring in $14$ colors. (I’m going to write a proof later.) But this not imply that $r>14$, because a $d$-separated subset $B$ of $A$ may be chosen by different method. So there is sense to look for subsets $A$ of lattice points with the biggest ratio $|A|/|B|$, where $B$ is a maximal $d$-separated subset of $A$.



                The following set has ratio $14$.



                ...
                ....
                ....
                ...


                Are there subsets of lattice points with the bigger ratio?







                share|cite|improve this answer












                share|cite|improve this answer



                share|cite|improve this answer










                answered Dec 5 '18 at 20:14









                Alex Ravsky

                39.2k32080




                39.2k32080























                    1














                    From the $ntimes n$ array, the following gives about $ n^2/15$, or one point in every fifteen.



                    $$x(3,3)+y(-1,4)$$






                    share|cite|improve this answer



















                    • 1




                      Do you think it's possible to achieve $n^2/14$? That was actually a bound I was hoping for to solve a problem which led me to this question. The problem says that it's always possible to choose a subset of a set of unit circles having area $S$ so that the area of a chosen subset is at least $2S/9$. If you use a well-known result saying that for a figure of area $S$ it's always possible to set a coordinate system so that the figure contains at least $S$ lattice points it quickly becomes clear that a bound of $n^2/14$ would solve the problem.
                      – mr. clock
                      Dec 4 '18 at 8:48
















                    1














                    From the $ntimes n$ array, the following gives about $ n^2/15$, or one point in every fifteen.



                    $$x(3,3)+y(-1,4)$$






                    share|cite|improve this answer



















                    • 1




                      Do you think it's possible to achieve $n^2/14$? That was actually a bound I was hoping for to solve a problem which led me to this question. The problem says that it's always possible to choose a subset of a set of unit circles having area $S$ so that the area of a chosen subset is at least $2S/9$. If you use a well-known result saying that for a figure of area $S$ it's always possible to set a coordinate system so that the figure contains at least $S$ lattice points it quickly becomes clear that a bound of $n^2/14$ would solve the problem.
                      – mr. clock
                      Dec 4 '18 at 8:48














                    1












                    1








                    1






                    From the $ntimes n$ array, the following gives about $ n^2/15$, or one point in every fifteen.



                    $$x(3,3)+y(-1,4)$$






                    share|cite|improve this answer














                    From the $ntimes n$ array, the following gives about $ n^2/15$, or one point in every fifteen.



                    $$x(3,3)+y(-1,4)$$







                    share|cite|improve this answer














                    share|cite|improve this answer



                    share|cite|improve this answer








                    edited Dec 4 '18 at 7:18

























                    answered Dec 4 '18 at 7:13









                    Empy2

                    33.4k12261




                    33.4k12261








                    • 1




                      Do you think it's possible to achieve $n^2/14$? That was actually a bound I was hoping for to solve a problem which led me to this question. The problem says that it's always possible to choose a subset of a set of unit circles having area $S$ so that the area of a chosen subset is at least $2S/9$. If you use a well-known result saying that for a figure of area $S$ it's always possible to set a coordinate system so that the figure contains at least $S$ lattice points it quickly becomes clear that a bound of $n^2/14$ would solve the problem.
                      – mr. clock
                      Dec 4 '18 at 8:48














                    • 1




                      Do you think it's possible to achieve $n^2/14$? That was actually a bound I was hoping for to solve a problem which led me to this question. The problem says that it's always possible to choose a subset of a set of unit circles having area $S$ so that the area of a chosen subset is at least $2S/9$. If you use a well-known result saying that for a figure of area $S$ it's always possible to set a coordinate system so that the figure contains at least $S$ lattice points it quickly becomes clear that a bound of $n^2/14$ would solve the problem.
                      – mr. clock
                      Dec 4 '18 at 8:48








                    1




                    1




                    Do you think it's possible to achieve $n^2/14$? That was actually a bound I was hoping for to solve a problem which led me to this question. The problem says that it's always possible to choose a subset of a set of unit circles having area $S$ so that the area of a chosen subset is at least $2S/9$. If you use a well-known result saying that for a figure of area $S$ it's always possible to set a coordinate system so that the figure contains at least $S$ lattice points it quickly becomes clear that a bound of $n^2/14$ would solve the problem.
                    – mr. clock
                    Dec 4 '18 at 8:48




                    Do you think it's possible to achieve $n^2/14$? That was actually a bound I was hoping for to solve a problem which led me to this question. The problem says that it's always possible to choose a subset of a set of unit circles having area $S$ so that the area of a chosen subset is at least $2S/9$. If you use a well-known result saying that for a figure of area $S$ it's always possible to set a coordinate system so that the figure contains at least $S$ lattice points it quickly becomes clear that a bound of $n^2/14$ would solve the problem.
                    – mr. clock
                    Dec 4 '18 at 8:48


















                    draft saved

                    draft discarded




















































                    Thanks for contributing an answer to Mathematics Stack Exchange!


                    • Please be sure to answer the question. Provide details and share your research!

                    But avoid



                    • Asking for help, clarification, or responding to other answers.

                    • Making statements based on opinion; back them up with references or personal experience.


                    Use MathJax to format equations. MathJax reference.


                    To learn more, see our tips on writing great answers.





                    Some of your past answers have not been well-received, and you're in danger of being blocked from answering.


                    Please pay close attention to the following guidance:


                    • Please be sure to answer the question. Provide details and share your research!

                    But avoid



                    • Asking for help, clarification, or responding to other answers.

                    • Making statements based on opinion; back them up with references or personal experience.


                    To learn more, see our tips on writing great answers.




                    draft saved


                    draft discarded














                    StackExchange.ready(
                    function () {
                    StackExchange.openid.initPostLogin('.new-post-login', 'https%3a%2f%2fmath.stackexchange.com%2fquestions%2f3023777%2fa-maximal-cardinality-subset-of-n-lattice-points-so-that-all-points-in-the-subse%23new-answer', 'question_page');
                    }
                    );

                    Post as a guest















                    Required, but never shown





















































                    Required, but never shown














                    Required, but never shown












                    Required, but never shown







                    Required, but never shown

































                    Required, but never shown














                    Required, but never shown












                    Required, but never shown







                    Required, but never shown







                    Popular posts from this blog

                    Different font size/position of beamer's navigation symbols template's content depending on regular/plain...

                    Berounka

                    I want to find a topological embedding $f : X rightarrow Y$ and $g: Y rightarrow X$, yet $X$ is not...